Đến nội dung

Hình ảnh

tuyển chọn các bài toán tính định thức

- - - - -

  • Please log in to reply
Chủ đề này có 14 trả lời

#1
sinh vien

sinh vien

    Thượng sĩ

  • Thành viên
  • 260 Bài viết

Sau đây là một số bài toán tính định thức theo các hướng độc đáo đã được sau trên các cuộc thi danh tiếng  (đề mục này sẽ được bổ sung dần dần )

   

Bài toán 1. (PUTNAM 2014)  Cho A là ma trận $n\times n$ trong đó phần tử ở hàng thứ i cột thứ j được cho bởi $\frac{1}{min(i,j)}$ với $1\leqslant i,j\leqslant n$ . Tính định thức của ma trận A

Lời giải. Từ điều kiện ở đầu ta thấy 

           A=$\begin{bmatrix} 1 & 1 &1 ...& 1& \\ 1& \frac{1}{2} &\frac{1}{2} ...& \frac{1}{2}& \\ 1 & \frac{1}{2}&\frac{1}{3} ...&\frac{1}{3} & \\1 &\frac{1}{2}&\frac{1}{3}...& \frac{1}{4}\\ ... & ... & ... &... \\ 1 & \frac{1}{2}&\frac{1}{3} ... & \frac{1}{n} & \end{bmatrix}$

   Để giải bài toán này ta sẽ áp dụng khai triển Laplace  cho hàng thứ n . Chú ý khi đó ma trận con thu được khi xóa hàng thứ n có dạng 

         \begin{bmatrix} 1 & 1& 1 & ... &1& 1\\1 & \frac{1}{2}&\frac{1}{2} & ...& \frac{1}{2} & \frac{1}{2}\\ ... & ... &... & ...&... & ...\\1 & \frac{1}{2}&\frac{1}{3} &..& \frac{1}{n-1} &\frac{1}{n-1} \end{bmatrix}$

Khi đó cột thứ n và cột thứ n-1 trùng nhau nên ma trận con chứa cả hai cột n và n-1 có định thức bằng 0 . Từ đó ta thấy

        $\begin{vmatrix} 1& 1 & 1 & ... &1 \\1 & \frac{1}{2} & \frac{1}{2} &... & \frac{1}{2}\\ 1& \frac{1}{2}&\frac{1}{2} &... &\frac{1}{2} \\ ...& ... &... & ... &... \\ 1& \frac{1}{2} &\frac{1}{3} & ... &\frac{1}{n} \end{vmatrix}=-\frac{1}{n-1}\begin{vmatrix} 1 & 1 & ... &1 \\ 1 & \frac{1}{2} &... &\frac{1}{2} \\... & ...& ... & ...\\ 1 & \frac{1}{2}&... &\frac{1}{n-1} \end{vmatrix}+\frac{1}{n}\begin{vmatrix} 1 & 1 & ... & 1\\1 & \frac{1}{2} & ... & \frac{1}{2}\\ ... &\... & ... &... \\ 1 & \frac{1}{2} &... &\frac{1}{n-1} \end{vmatrix}$

    Nếu đặt $D_{n}=det(A)$ trong đó A có cấp n thì $D_{n}=\left ( \frac{1}{n} -\frac{1}{n-1}\right ) D_{n}=\frac{-1}{n(n-1)}D_{n-1}$

    Sử dụng hệ thức truy hồi này và chú ý $D_{1}=1$ ta được

$D_{n}=\frac{-1}{n(n-1)}\frac{-1}{(n-1)(n-2)}...\frac{-1}{3.2}\frac{-1}{2.1}=\frac{(-1)^{n+1}}{n!(n-1)!}$

   Bài toán 2 (ĐHBK -2013) .Cho $x_{i},y_{i},1\leq i\leq n$ là các số phức với $x_{i}y_{j}$$\neq 1$ với mọi cặp $x_{i},y_{j}$.

  Tính định thức của ma trận $M=(m_{ij})_{n\times n}$ tróng đó $m_{ij}=\frac{1}{1-x_{i}y_{j}}$

lời giải. Để cho thuận tiện ta quy ước $D_{y_{1}y_{2}...y_{n}}^{x_{1}x_{2}...x_{n}}=det(M)$. Ta thấy

 n=2 : $D_{y_{1}y_{2}}^{x_{1}x_{2}}=\begin{vmatrix} \frac{1}{1-x_{1}y_{1}} &\frac{1}{1-x_{1}y_{2}} \\ \frac{1}{1-x_{2}y_{1}} &\frac{1}{1-x_{2}y_{2}} \end{vmatrix}=\frac{(x_{1}-x_{2})(y_{1}-y_{2})}{(1-x_{1}y_{1})(1-x_{1}y_{2})(1-x_{2}y_{1})(1-x_{2}y_{2})}$  ( tính toán tương đối đơn giản nên mình không nêu ra cụ thể )

Ta có thể tính trực tiếp thêm một số giá trị của n . Dự đoán :

$D_{y_{1}...y_{n}}^{x_{1}...x_{n}}=\frac{\prod_{1\leqslant i< j\leqslant n}(x_{i}-x_{j})(y_{i}-y_{j})}{\prod_{1\leq i,j\leq n}(1-x_{i}y_{j})}$. Ta sẽ chứng minh quy nạp công thức này.

   Áp dụng khai triển Laplace cho cột thứ nhất ta được

$D_{y_{1}..y_{n+1}}^{x_{1}...y_{n+1}}=\sum_{i=1}^{n+1}(-1)^{i+1}\frac{1}{1-x_{i}y_{1}}D_{y_{2}...y_{n+1}}^{x_{1}...x_{i-1}x_{i+1}...x_{n+1}}$

$=\sum_{i=1}^{n+1}(-1)^{i-1}\frac{1}{1-x_{i}y_{1}}\frac{\prod_{1\leq k< l\leq n+1,k,l\neq i}(x_{k}-x_{l})\prod_{2\leq k< l\leq n+1}(y_{k}-y_{l})}{\prod_{1\leq k\leqslant n+1,k\neq i,2\leq l\leqslant n+1}(1-x_{k}y _{l})}$

$\sum_{i=1}^{n+1}(-1)^{i-1}\prod_{k=1,k\neq i}^{n+1}(1-x_{k}y_{1})\frac{\prod_{1\leqslant k< l\leq n+1}(x_{k}-x_{l})\prod_{k=2}^{n+1}(1-x_{i}y_{k})\prod_{2\leq k< l\leq n+1}(y_{k}-y_{l})}{\prod_{k=1}^{i-1}(x_{k}-x_{i})\prod_{k=i+1}^{n+1}(x_{i}-x_{k})\prod_{1\leqslant k,l\leqslant n+1}(1-x_{k}y_{l})}$

$=\left ( \sum_{i=1}^{n+1}\prod_{k=2}^{n+1}(1-x_{i}y_{k})\prod_{k=1,k\neq i}^{n+1} \frac{1-x_{k}y_{1}}{x_{i}-x_{k}}\right )\frac{\prod_{1\leq k< l\leq n+1}(x_{k}-x_{l})\prod_{2\leq k< l\leq n+1}(y_{k}-y_{l})}{\prod_{1\leqslant k,l\leq n+1}(1-x_{k}y_{l})}$

 Đặt $P(x)=\prod_{k=2}^{n+1}(1-xy_{k})$ thì $P(x)$ là một đa thức bậc n và $P(x_{i})=\prod_{k=2}^{n+1}(1-x_{i}y_{k}),i=1,2,..n+1$ . Áp dụng công thức Lagrange ta được:

  $P(x)=\sum_{i=1}^{n+1}\prod_{k=2}^{n+1}(1-x_{i}y_{k})\prod_{k=1,k\neq i}^{n+1}\frac{x-x_{k}}{x_{i}-x_{k}}$

 Thay $x=\frac{1}{y_{1}}$ , nhân hai vế cho $y_{1}^{n}$ ta được

      $\prod_{k=2}^{n+1}(y_{1}-y_{k})=\sum_{i=1}^{n+1}\prod_{k=2}^{n+1}(1-x_{i}y_{k})\prod_{k=1,k\neq i}^{n+1}\frac{1-x_{k}y_{1}}{x_{i}-x_{k}}$.

  Từ đây ta dễ dàng thu được kết quả mong muốn.


Bài viết đã được chỉnh sửa nội dung bởi WhjteShadow: 22-06-2015 - 10:20


#2
sinh vien

sinh vien

    Thượng sĩ

  • Thành viên
  • 260 Bài viết

Bài toán:(Putnam 2009 )Cho n > 3, n nguên dương. Tính định thức:

 $D_{n}=\begin{vmatrix} cos1 &cos2 &... &cosn \\ cos(n+1) & cos(n+2) &... &cos2n \\... &... &... &... \\cos(n^{2}-n+1) & cos(n^{2}-n+2) &... &cosn^{2} \end{vmatrix}$

Lời giải.  Lấy cột thứ 3 + cột thứ 1$\rightarrow$ cột thứ 1 và sử dụng biến đổi lượng giác , ta được :

$D_{n}=\begin{vmatrix} 2cos2cos1 &cos2 &... & ... &... cos(n) \\2cos(n+2)cos1 &cos(n+2) &... & ... &cos2n \\... & ... & & ... &... \\... &... & ... & ... &... \\2cos(n^{2}-n+2)cos1 & cos(n^{2}-n+2) &... &... & cosn^{2} \end{vmatrix}$

   Lúc này cột thứ 1= 2cos1 cột thứ 2 $\Rightarrow$ det$D_{n}=0$.

Bài toán . Xét các số phức$z_{1},z_{2},...,z_{2n}$ thỏa mãn điều kiện  $\left | z_{1} \right |=\left | z_{2} \right |=...=\left | z_{n+3} \right |$ và $argz_{1}\geqslant argz_{2}\geqslant ...\geq argz_{n+3}$.

  Tính định thức của ma trận B ,trong đó  $b_{ij}=\left | z_{i}-z_{j+n} \right |,i,j\in \left \{ 1,2,...n \right \}$.

 Lời giải . Ta có nhận xét sau bài toán đề cập đến modun và argument của một số phức nên ta sẽ sử dụng dạng lượng giác của số phức trong các tính toán. 

    Với hai số phức $z=r(cosx+isinx);\omega =r(cosy+isiny)\Rightarrow \left | z-\omega \right |=2r\left | sin\left ( \frac{x-y}{2} \right ) \right |$. 

  Kết hợp với điều kiện ở đầu bài, ta thấy B có dạng như sau:

$B=(2r)^{n}\begin{vmatrix} sin(x_{1}-x_{n+1}) &sin(x_{1}-x_{n+2}) & sin(x_{1}-x_{n+3}) & ...\\... &... & ... &... \\ sin(x_{n}-x_{n+1}) & sin\left ( x_{n}-x_{n+2} \right ) & sin(x_{n}-x_{n+3}) & ... \end{vmatrix}$

 trong đó $x_{i}$ kí hiệu tương ứng argument của $z_{i}$ và r=$\left | z_{1} \right |=\left | z_{2} \right |=...=\left | z_{2n} \right |$

 Nếu ta tách det B theo các cột thì dễ dàng nhận thấy det B=0 

 

 



#3
sinh vien

sinh vien

    Thượng sĩ

  • Thành viên
  • 260 Bài viết

Bài toán:(Putnam 2009 )Cho n > 3, n nguên dương. Tính định thức:

 $D_{n}=\begin{vmatrix} cos1 &cos2 &... &cosn \\ cos(n+1) & cos(n+2) &... &cos2n \\... &... &... &... \\cos(n^{2}-n+1) & cos(n^{2}-n+2) &... &cosn^{2} \end{vmatrix}$

Lời giải.  Lấy cột thứ 3 + cột thứ 1$\rightarrow$ cột thứ 1 và sử dụng biến đổi lượng giác , ta được :

$D_{n}=\begin{vmatrix} 2cos2cos1 &cos2 &... & ... &... cos(n) \\2cos(n+2)cos1 &cos(n+2) &... & ... &cos2n \\... & ... & & ... &... \\... &... & ... & ... &... \\2cos(n^{2}-n+2)cos1 & cos(n^{2}-n+2) &... &... & cosn^{2} \end{vmatrix}$

   Lúc này cột thứ 1= 2cos1 cột thứ 2 $\Rightarrow$ det$D_{n}=0$.

Bài toán . Xét các số phức$z_{1},z_{2},...,z_{2n}$ thỏa mãn điều kiện  $\left | z_{1} \right |=\left | z_{2} \right |=...=\left | z_{n+3} \right |$ và $argz_{1}\geqslant argz_{2}\geqslant ...\geq argz_{n+3}$.

  Tính định thức của ma trận B ,trong đó  $b_{ij}=\left | z_{i}-z_{j+n} \right |,i,j\in \left \{ 1,2,...n \right \}$.

 Lời giải . Ta có nhận xét sau bài toán đề cập đến modun và argument của một số phức nên ta sẽ sử dụng dạng lượng giác của số phức trong các tính toán. 

    Với hai số phức $z=r(cosx+isinx);\omega =r(cosy+isiny)\Rightarrow \left | z-\omega \right |=2r\left | sin\left ( \frac{x-y}{2} \right ) \right |$. 

  Kết hợp với điều kiện ở đầu bài, ta thấy B có dạng như sau:

$B=(2r)^{n}\begin{vmatrix} sin(x_{1}-x_{n+1}) &sin(x_{1}-x_{n+2}) & sin(x_{1}-x_{n+3}) & ...\\... &... & ... &... \\ sin(x_{n}-x_{n+1}) & sin\left ( x_{n}-x_{n+2} \right ) & sin(x_{n}-x_{n+3}) & ... \end{vmatrix}$

 trong đó $x_{i}$ kí hiệu tương ứng argument của $z_{i}$ và r=$\left | z_{1} \right |=\left | z_{2} \right |=...=\left | z_{2n} \right |$

 Nếu ta tách det B theo các cột thì dễ dàng nhận thấy det B=0 

 

 



#4
sinh vien

sinh vien

    Thượng sĩ

  • Thành viên
  • 260 Bài viết

  Lưu ý với các bạn bài toán này mình đã thay đổi yêu cầu để cho đơn giản bạn có thể tìm thấy nội dung đầy đủ của nó ở Putnam 1984

 Bài toán . Cho n là một số nguyên dương. Gỉa sử a,b,x là các số thực , với $a^{2}\neq b^{2}$, Ma trận vuông $M_{n}$ cấp 2n trong đó phần tử (i,j) được xác định:

$m_{ij}=\left\{\begin{matrix} x & i=j\\ a & i\neq j &i+j =2k \\ b & i\neq j & i+j=2k+1 \end{matrix}\right.$ 

  Tính $detM_{n}$ theo a,b,n

Lời giải. Gọi N là ma trận $M_{n}$ khi x =a . Từ định nghĩa ta dễ dàng nhận thấy N có hạng bằng 2 ( chỉ có hai hàng phân biệt ). Do đó 0 là một giá trị riêng của N với số bội bằng 2n-2.

  Nếu gọi e là ma trận $2n\times 1$ với các phần tử bằng một . 

  Với một chút tính toán ta sẽ thấy Ne=n(a+b)e- điều này có nghĩa là n(a+b) là một giá trị riêng khác của N ứng với vector riêng e

   Ta thấy vết của N bằng 2an $\Rightarrow$ Tồn tại một giá trị riêng thứ ba bằng 2an-n(a+b)=n(a-b)

 Từ đây dễ dàng suy ra rằng

$det(N-\lambda I_{2n})=\lambda ^{2n-2}(\lambda -n(a+b))(\lambda -n(a-b))$

  Để giải quyết trọn vẹn bài toán ta chú ý một điểm quan trọng sau $M_{n}=N-(a-x)I_{2n}$

  Thay $\lambda =a-x$ từ công thức trên ta suy ra

 $detM_{n}=(a-x)^{2n-2}(a-x-n(a-b))(a-x+n(a-b))$.

 



#5
sinh vien

sinh vien

    Thượng sĩ

  • Thành viên
  • 260 Bài viết

Bài toán. Cho ma trận vuông A cấp n , với $a_{ij}=(i,j)$, trong đó (i,j) là ước chung lớn nhất của i và j

Tính det A


Bài viết đã được chỉnh sửa nội dung bởi sinh vien: 19-05-2015 - 10:07


#6
sinh vien

sinh vien

    Thượng sĩ

  • Thành viên
  • 260 Bài viết

Bài toán ( Sydney-2013). Tính định thức của ma trận vuông cấp n trong đó phần tử (i,j) bằng i nếu $i\neq j$ và bằng i+1 nếu i=j

Lời giải. Gọi $M_{n}$ là ma trận được định nghĩa, $I_{n}$ là ma trận đơn vị cấp n.

Khi đó ta thấy từ định nghĩa thì ma trận $M_{n}-I_{n}$ có hàng thứ i là (i,i,...,i) nên có hạng bằng 1 và định thức bằng 0 .

 Suy ra 1 là một giá trị riêng của ma trận $M_{n}$ có số bội bằng n-1.

Ta thấy vết của ma trận $M_{n}$ bằng $\sum_{i=1}^{n}(i+1)=\frac{n^{2}+3n}{2}$ nên giá trị riêng còn lại của ma trận $M_{n}$ bằng $\frac{n^{2}+3n}{2}-(n-1)=\frac{n^{2}+n+2}{2}$

  Do đó : $detM_{n}=1^{n-1}\times\frac{n^{2}+n+2}{2}=\frac{n^{2}+n+2}{2}$


Bài viết đã được chỉnh sửa nội dung bởi sinh vien: 21-05-2015 - 16:38


#7
sinh vien

sinh vien

    Thượng sĩ

  • Thành viên
  • 260 Bài viết

Bài toán ( Sydney-2009 ) Cho $A_{n}$ là ma trận vuông cấp n trong đó phần tử (i,j) bằng 1 nếu $n\leq i+j\leq n+1$ và bằng 0 trong các trường hợp còn lại.

 Tìm các giá trị riêng của $A_{n}$



#8
sinh vien

sinh vien

    Thượng sĩ

  • Thành viên
  • 260 Bài viết

Bài toán ( Sydney-2005). Tính định thức của ma trận vuông A cấp n trong đó

                                                             $a_{ij}=\frac{(2i+2j-2)!}{2^{2i+2j-2}(i+j-1)!}$

Lời giải. Ta chứng minh kết quả sau:

  Cho  $\alpha \in \mathbb{R}$ , giả sử $C=(c_{ij})_{i,j=1}^{n}$ thỏa $c_{i,j+1}=(i+j-\alpha )c_{ij}$

 với mọi $1\leq i\leq n,1\leq j\leq n-1$ thì $detC=\prod_{i=1}^{n}(i-1)!c_{i,1}$



#9
sinh vien

sinh vien

    Thượng sĩ

  • Thành viên
  • 260 Bài viết

Bài toán ( Nordic-2011) Chứng minh rằng

$\begin{vmatrix} a & b & 0 & 0 & 0 & ... & 0 & 0 &0 \\ c &a &b & 0 & 0 & ... & 0 &0 &0 \\0 & c & a & b &0 & ... & 0 & 0 &0 \\... & ... & ... &... &... &... &... &... &... \\ 0 & 0 & 0 & 0 &0 & ... & a &b &0 \\0 & 0 & 0 & 0 & 0 & ... & c & a &b \\ 0 & 0 & 0 & 0 & 0 &... &0 &c & a \end{vmatrix}_{n\times n}=\prod_{k=1}^{n}\left ( a-2\sqrt{bc}cos\frac{k\pi }{n+1} \right )$



#10
sinh vien

sinh vien

    Thượng sĩ

  • Thành viên
  • 260 Bài viết

Lời giải. Ta sẽ tính định thức bằng các xác định các giá trị riêng của ma trận.

  Trước tiên để cho thuận tiện trong các trình bày ta sẽ xét một trường hợp đặc biệt khi a=0 và bc=1

kí hiệu là $D_{n}$ . Đặt $P_{n}(\lambda )=det(\lambda I_{n}-D_{n})$ .

  Áp dụng khai triển Laplace liện tiếp cho ma trận $\lambda I_{n}-D_{n}$ , lần thứ nhất theo cột thứ nhất , lần thứ hai theo hàng thứ nhất, ta thu được hệ thức

$P_{n}(\lambda )=(-1)^{1+1}\lambda P_{n-1}(\lambda )+(-1)^{2+1}(-c) (-1)^{1+1}(-b)P_{n-2}(\lambda )=\lambda P_{n-1}(\lambda )-P_{n-2}(\lambda )$

  Ta qui ước $P_{0}(\lambda )=1$, bằng tính toán trực tiếp $P_{1}(\lambda )=1$ nên với mọi $n\geq 2$ ta luôn có

                     $P_{n}(\lambda )=\lambda P_{n-1}(\lambda )-P_{n-2}(\lambda )$

 Bằng phương pháp quy nạp ta dễ dàng nhận thấy;

                                 $P_{n}(2cos\theta )=\frac{sin(n+1)\theta )}{sin\theta }, 0< \theta < \pi$

  Dễ dàng nhận thấy $P_{n}(\lambda _{k})=0$ với $\lambda _{k}=2cos\frac{k\pi }{n+1},k=1,2,...n$  mà $degP_{n}(x)=n$ . Từ đây suy ra $\left \{ 2cos\frac{k\pi }{n+1} \right \}_{k=1}^{n}$ là tất cả các giá trị riêng của $D_{n}$.

   Trong trường hợp tổng quát ta thấy $A_{n}=aI_{n}+\sqrt{bc}\overline{D_{n}}$, trong đó $A_{n}$ là ma trận cho trong đề bài còn $\overline{D_{n}}$ có các phần tử trên đường chéo chính bằng 0 và thỏa mãn các tính chất trong trường hợp riêng mà ta đã khảo sát. Theo kết quả trên ta thấy $\left \{ a+2\sqrt{bc}cos\frac{k\pi }{n+1} \right \}_{k=1}^{n}$ là các giá trị riêng của $A_{n}$

   Do đó

       $det(A_{n})=\prod_{k=1}^{n}\left ( a+2\sqrt{bc}cos\frac{k\pi }{n+1} \right )=\prod_{k=1}^{n}\left ( a-2\sqrt{bc}cos\frac{(n+1-k)\pi }{n+1} \right )=\prod_{k=1}^{n}\left ( a-2\sqrt{bc}cos\frac{k\pi }{n+1} \right )$


Bài viết đã được chỉnh sửa nội dung bởi sinh vien: 22-05-2015 - 14:31


#11
sinh vien

sinh vien

    Thượng sĩ

  • Thành viên
  • 260 Bài viết

Bài toán ( Sydney-2011) Cho m, n là hai số nguyên dương sao cho $m\geq n$ Gọi A là ma trậ vuông cấp n sao cho phần tử (i,j) bằng $C_{mj}^{i}$ . Tính det A .

    Đáp số : $detA=m^{\frac{n(n+1)}{2}}$



#12
sinh vien

sinh vien

    Thượng sĩ

  • Thành viên
  • 260 Bài viết

Bài toán ( ĐH-FPT 2013 ) Tính định thức sau

         $\begin{vmatrix} x+a_{1} &a_{2} & ... &a_{n} \\a_{1} & x+a_{2} & ... & a_{n}\\ ... &... &... &... \\a_{1} & a_{2} &... & x+a_{n} \end{vmatrix}$

Cách 1 ( Biến đổi sơ  cấp )  Cộng  tất cả các cột 2 ,3,...,n vào cột đầu tiên ta thu được

                                 $\begin{vmatrix} x+a_{1}+a_{2}+...+a_{n} & a_{2} &... & a_{n}\\ x+a_{1}+a_{2}+...+a_{n}&x+a_{2} &... &a_{n} \\ ...& ....& ... &.... \\x+a_{1}+a_{2}+...+a_{n} &a_{2} &... &x+a_{n} \end{vmatrix}$

 Tiếp theo trừ hàng thứ 1 cho các hàng 2,3,...n ta được

                  $\begin{vmatrix} x+a_{1}+a_{2}+...+a_{n} & a_{2} & ... & a_{n}\\ 0 & x & ... & a_{n}\\... & ... & ... & ...\\ 0 & 0 & ... & x \end{vmatrix}$

  Đến đây dễ  thấy giá trị định thức cần tìm bằng $x^{n-1}(x+a_{1}+a_{2}+...+a_{n})$

Cách 2 ( Lý thuyết về giá trị riêng - đa thức đặc trưng )  Đặt  $P(\lambda )=det(A-\lambda I_{n})$,

trong đó $A=\begin{bmatrix} a_{1} &a_{2} & ... &a_{n} \\ a_{1} & a_{2} & ... &a_{n} \\ ...& ... & ... &... \\a_{1} & a_{2}& ... & a_{n} \end{bmatrix}$. 

  Ta thấy rank A=1 , nên 0 là một  giá trị riêng của ma trận A và có số bội bằng $n-rankA=n-1$. 

  Theo đó giá trị riêng còn lại của ma trận A  bằng $a_{1}+a_{2}+...+a_{n}$ . Lưu ý là hệ  số của $\lambda ^{n}$ trong $P(\lambda )$ là $(-1)^{n}$

nên $P(\lambda )=det(A-\lambda I_{n})(-1)^{n}\lambda ^{n-1}(\lambda +a_{1}+a_{2}+...+a_{n})$ 

 Thay $\lambda =-x$ thay thấy $det(A+xI_{n})=x^{n-1}(x+a_{1}+a_{2}+...+a_{n})$.

  Chú ý vế trái là định thức ta cần tính.

      



#13
sinh vien

sinh vien

    Thượng sĩ

  • Thành viên
  • 260 Bài viết

Tiếp theo là một số bài toán tính toán định thức có trên tạp chí American Mathematical ....

Bài toán ( AMM-11179) Cho các số nguyên dương i, j đặt $m_{i,j}=\left\{\begin{matrix} -1 &i+1\equiv 0(mod j) \\0 & i+1\not\equiv 0(modj) \end{matrix}\right.$.Gỉa sử $M_{n}$ là ma trận vuông cấp n-1 có phần tử (i,j) là $m_{i,j}$.

  Chứng minh rằng $det(M_{n})=\mu (n)$, trong đó $\mu$ là hàm Mobius . ( Tạp chí Epsilon)


Bài viết đã được chỉnh sửa nội dung bởi sinh vien: 02-06-2015 - 07:22


#14
sinh vien

sinh vien

    Thượng sĩ

  • Thành viên
  • 260 Bài viết

Tiếp theo chúng ta sẽ ôn tập lại phương pháp giá trị riêng thông qua một ví dụ nhỏ.

Bài toán ( Saint Peterburg -2007) Cho ma trận $M=(m_{ij})_{n\times n},$ trong đó $m_{ij}=\begin{cases} a_{i}a_{j} & \text{ }i\neq j \\ a_{i}^{2}+k& \text{ if } i= j \end{cases}$.

Tính detM.

Lời giải. 

Đặt: $A=\begin{pmatrix} a_{1}^{2} &a_{1} a_{2} & ... & a_{1}a_{n}\\ a_{2}a_{a} &a_{2}^{2} &... & a_{2}a_{n}\\ ...& ... &... &... \\a_{n}a_{1} &a_{n}a_{2} & ... &a_{n}^{2} \\ & & & \end{pmatrix}$

thì khi đó detM=det(A+kE), trong đó E là ma trận đơn vị cấp n.

  Dễ dàng nhận thấy rankA=1 do đó 0 là một giá trị riêng của ma trận A và có số bội là n-1 nên giá trị riêng còn lại sẽ là $a_{1}^{2}+a_{2}^{2}+...+a_{n}^{2}$ nên ta suy ra đẳng thức bên dưới đây

              $det(A-\lambda E)=(-1)^{n}\lambda ^{n-1}(\lambda -(a_{1}^{2}+a_{2}^{2}+...+a_{n}^{2}))$.

Thay $\lambda =-k$ ta sẽ thu được đáp án cho câu hỏi ban đầu là $k^{n-1}(k+a_{1}^{2}+a_{2}^{2}+...+a_{n}^{2})$.

 Bên dưới đây là một file đề thi bằng tiếng Nga dành cho các bạn nghiên cứu thêm

 File gửi kèm  2007.pdf   172.63K   183 Số lần tải


Bài viết đã được chỉnh sửa nội dung bởi sinh vien: 19-06-2015 - 09:26


#15
sinh vien

sinh vien

    Thượng sĩ

  • Thành viên
  • 260 Bài viết

Bài toán (AMM11270) Gọi $S_{n}$ là ma trận vuông cấp n có các phần tử thuộc tập $\left \{ 1,2,...,n^{2} \right \}$ .Các phần tử được sắp xếp theo hình xoắn ốc theo chiều tăng của các giá trị.

Tính $detS_{n}$.

Đáp số: $detS_{n}=(-1)^{\frac{n(n-1)}{2}}4^{n-1}\frac{3n-1}{2}\prod_{k=0}^{n-2}\left ( k+\frac{1}{2} \right )$

File lời giải:

 File gửi kèm  AMM11270.pdf   63.73K   141 Số lần tải


Bài viết đã được chỉnh sửa nội dung bởi sinh vien: 19-06-2015 - 09:42





1 người đang xem chủ đề

0 thành viên, 1 khách, 0 thành viên ẩn danh